Guys how do I solve 3x+x. PLEASE HELP!!!!!

Answers

Answer 1

Answer: 4x

Step-by-step explanation:

Combine like terms

3x + x

4x

Hope this helps!

Answer 2

Answer:

  4x

Step-by-step explanation:

You want to "solve" 3x +x.

Coefficient

Start by understanding the meaning of 3x. The coefficient of 3 tells you how many instances of x you have. The term 3x is a representation of repeated addition:

  3x = x + x + x . . . . . . . . x is included in the sum 3 times

The sum 3x + x can be written out as ...

  3x + x = (x + x + x) + x

This is fully equivalent to ...

  3x + x = x + x + x + x

We recognize the sum on the right includes 4 instances of x, so can be written using a coefficient as ...

  x + x + x + x = 4x

This means ...

  3x + x = 4x

Distributive property

If we use the distributive property, we can also get there this way:

  3x + x = 3x + 1x = (3 +1)x = 4x

<95141404393>


Related Questions

mathematics and statistics teacher rex boggs, from australia, weighed the bar of soap in his shower stall before showering in the morning. the data on day and soap weight (in grams) appear in the table. (notice that he forgot to weigh the soap on some days!) here are the data.

Answers

As per the concept of linear regression, the equation of the data is y ​= 133.1807 − 6.3096x.

The term linear regression means to determine the character and strength of the association between a dependent variable and a series of other independent variables.

Here we have given that mathematics and statistics teacher rex boggs, from Australia, weighed the bar of soap in his shower stall before showering in the morning. And the data on day and soap weight (in grams) appear in the table.

Here we need to find the linear regression equation that represents the given situation.

Based on the concept of linear regression, we know the the general formula for the regression is written as,

=> y = a + bx

FRom the given data, we have identified the value of the functions as,

a = 133.1807

b = -6.3096

r² = 0.9962

r = -0.9981

When we apply these values on the formula, then we get the equation as,

Then it implies the regression line is,

=> ŷ = a + bx = 133.18076.3096x

here a represents the day and y represents the weight in grams.

To know more about regression here

https://brainly.com/question/14184702

#SPJ4

Suppose that survey is planned to estimate the proportion of population that is left-handed The sample data will be used to form confidence interval: Which one of the following combinations of sample size and confidence leve will give the widest interval? n E 500, confidence level 909 n = 1,000, confidence level 909 n = 500, confidence level 959 n = 1,000, confidence level 959 Explain why this sample size and confidence level will give the widest interval: The Select-- sample size and Select- confidence level will both tend to increase the width of the confidence interval:

Answers

The 500 sample size and 95% confidence level will both tend to increase the width of the confidence interval.

In this question, we have been given  the combinations of sample size and confidence level.

We need to select a combinations of sample size and confidence level that will give the widest interval.

We know that the confidence level is typically set in the range of 99% to 80%. The 95% confidence interval will be wider than the 90% interval and which will be wider than the 80% interval.

As we know increasing the confidence will increase the margin of error which results in a wider interval.

But increasing the sample size decreases the width of confidence intervals, as it decreases the standard error.

This means, for the widest interval we need to select a high confidence interval and small sample size.

Therefore, the sample size = 500 and the confidence interval = 95% will give the widest interval.

Learn more about the confidence interval here:

https://brainly.com/question/22851322

#SPJ4

Lance Industries borrowed $130,000. The company plans to set up a sinking fund that will repay the loan at the end of 18 years. Assume a 6% interest rate compounded semiannually. What amount must Lance Industries pay into the fund each period? (Round your answer to the nearest cent.)

Answers

The amount must Lance Industries pay into the fund each period is $2,054.49

Define Sinking Funds

A fund set aside to cover a certain future expense is known as a sinking fund. The sinking fund can be thought of as an annuity if equal contributions are made on a regular basis; its value will rise until it is sufficient to pay the future expense.

Given,

Lance Industries borrowed = $130,000

Time = 18 years

Interest rate = 6%

The future value is,

FVₙ =PMT( (1+i/m)ⁿ - 1 ) / (i/m) )

PMT = payment fund semiannually.

i = 6% or 0.06

m = 2  (semiannually will be 2 times per year)

n = 18 * 2 = 36 periods

FV₃₆ = $130,000

Now, plug in the values in formula

FVₙ =PMT( (1+i/m)ⁿ - 1 ) / (i/m) )

130,000 = PMT ( (1 + 0.06/2)³⁶ - 1 ) / (0.06/2) )

130,000 = PMT ( (1.03)³⁶ - 1 ) / (0.03) )

PMT = 130,000 * ( 0.03 / (1.03)³⁶ - 1 ) )

        = $2,054.49

Hence, the amount must Lance Industries pay into the fund each period is $2,054.49.

To read more about Sinking Funds.

https://brainly.com/question/26361248

#SPJ1

How much would you need to deposit in an account now in order to have $5,000.00 in the account in 821 days?

Assume the account earns 5 % simple interest.

You would need to deposit ___
in your account now.

Answers

The principal investment required to get an accrued amount of $5,000.00 in 821 days is  $4,494.52.

What is the principal need for the investment?

Simple interest is expressed as;

A = P(1 + rt)

Where A is accrued amount, P is principal, r is interest rate and t is time.

Given that;

Accrued amount A = $5,000.00Elapsed time t = 821 days = 821/365 yrsInterest rate r = 5% = 5/100 = 0.05Principal P = ?

Plug the given values into the above formula and solve for principal P.

A = P(1 + rt)

P = A / (1 + rt)

P = $5,000.00 / ( 1 + ( 0.05 × 821/365 ) )

P = $5,000.00 / ( 1 + 821/7300 )

P = $5,000.00 / ( 8121/7300 )

P = $4,494.52

Therefore, the principal required is $4,494.52.

Learn more about simple interest here: brainly.com/question/25845758

#SPJ1

find the least integer n such that n! is divisible by[tex]10^{6}[/tex]

Answers

Answer:

25!

in this type of questions you need to find 10 makers ( it's means you need to find numbers which has 2 or 5 in their prime factors) obviously there is a bunch of numbers which has 2 in their prime factors.

so we need to find multiples of 5. But you should be aware that some numbers have more than one prime factors of 5 e.g. 25 .

ok let's go back to the question.

starting from 1 , we've got 5, 10 , 15 ,20 &25(5²)

so the answer is 25!

Let P (t) represent the amount of _ certain drq milligrams; that = he bloodstream time hou? The rate at which the= drug leaves the bloodstream proportional to the amount of the drug bloodstream: The amount of the drug in the bloodstream can be modeled by # KP ; where constant and- time; hours At time 500 milligrams of the drug was administered. At the moment when the amount of the drug in the bloodstream 50 milligrams, the amount decreasing at a rate milligrams per hour Which of the following an expression for P (t) P (t) = 500 0.23t P (t) = 500 4t P (t) = s0e 0.23 P (t) = e-02x + 499

Answers

As proportional relationship, the amount decreasing at a rate milligrams per hour expression P(t) = 500 + e⁻⁰²ˣ

Proportional relationship:

In math, the relationships between two variables where their ratios are equivalent is known as proportional relationship.

Given,

Here we have the rate at which the= drug leaves the bloodstream proportional to the amount of the drug bloodstream: The amount of the drug in the bloodstream can be modeled by K; where constant and- time; hours At time 500 milligrams of the drug was administered. At the moment when the amount of the drug in the bloodstream 50 milligrams, the amount decreasing at a rate milligrams per hour.

Here we need to find the expression.

While we looking into the given question, we have identified the following details,

=> Bloodstream = 500 milligrams

Here they state that the bloodstream 50 milligrams, the amount decreasing at a rate milligrams per hour.

So, the expression for this proportional relationship is written as,

=> P(t) = 500 + e⁻⁰²ˣ

To know more about Proportional relationship here.

https://brainly.com/question/29765554

#SPJ4

The stock of WAL-MAT pays a dividend of $.88. The stock opened at $18.25 and closed at $18.33. The stock yield is:
Multiple Choice
4.8%
4.2%
4.1%

Answers

The stock yield for the stocks will be 4.8%. The correct option is A.

What is an expression?

The mathematical expression combines numerical variables and operations denoted by addition, subtraction, multiplication, and division signs.

Mathematical symbols can be used to represent numbers (constants), variables, operations, functions, brackets, punctuation, and grouping. They can also denote the logical syntax's operation order and other properties.

To get the stock to yield use this formula - annual dividend / current stock price or share price

Substitute the dividend of $.88 and the closing price of $18.33 to the formula above. Divide and simplify.

Stock yield = .88 / 18.33

Stock yield = 0.0480 x 100

Stock yield = 4.8%

Therefore, the stock yield for the stocks will be 4.8%. The correct option is A.

To know more about an expression follow

https://brainly.com/question/15055460

#SPJ1

A cement mix calls for 2 cups of water and 1 1/4 pounds of mix to make five large tiles stick to the ground, Yolanda has plenty of water but only 4 3/8 pounds of mix. How many large tiles (including parts of a tile) can she stick to the ground? You MUST think conceptually and illustrate your thinking with a model. Do NOT use proportional reasoning.

Answers

175/26 large tiles can she stick to the ground.

What is Equation?

Two or more expressions with an Equal sign is called as Equation.

Given,

A cement mix calls for 2 cups of water and [tex]1\frac{1}{4}[/tex] pounds of mix to make five large tiles stick to the ground,

2+ [tex]1\frac{1}{4}[/tex]=5

2+5/4

13/4

Which means 13/4 is needed for 5 large tiles.

Yolanda has plenty of water but only [tex]4\frac{3}{8}[/tex] pounds of mix

We need to find how many large tiles can she stick to the ground.

Let x be the number of tiles.

[tex]4\frac{3}{8}[/tex] =35/8

So

(13/4)/5=(35/8)/x

Apply cross multiplication

13/4x=35/8×5

13/4x=175/8

x=175/8×4/13

x=700/104=350/52=175/26

Hence, 175/26 large tiles can she stick to the ground.

To learn more on Equation:

https://brainly.com/question/10413253

#SPJ1


Find the sum of following two quantities if the number is reprensented by x
i. Seven multiplied by the sum of nine and a number.
ii. The sum of four and the number.

Answers

The sum of two (63 + 7x) and (4 + x) quantities will be 8x + 67.

What is Algebra?

Algebra is the study of abstract symbols, while logic is the manipulation of all those ideas.

The acronym PEMDAS stands for Parenthesis, Exponent, Multiplication, Division, Addition, and Subtraction. This approach is used to answer the problem correctly and completely.

Let 'x' be the number.

The first number is given as,

⇒ 7(9 + x)

⇒ 63 + 7x

The second number is given as,

⇒ 4 + x

Then the sum of two numbers is given as,

⇒ 63 + 7x + 4 + x

⇒ 8x + 67

The sum of two (63 + 7x) and (4 + x) quantities will be 8x + 67.

More about the Algebra link is given below.

https://brainly.com/question/953809

#SPJ1

find the indefinite integral using integration by parts with the given choices of u and dv. x2 ln(x) dx;

Answers

Indefinite integral using integration by parts from x2 ln(x) dx is

[tex]\frac{1}{3}[/tex] [tex]X^{3\\}[/tex] (ln(x) - [tex]\frac{1}{3}[/tex]) + C

The given choice of u and dv

[tex]x^{2} ln(x) dx[/tex]

we can use the formula

∫ u dv = uv - ∫ v du

Then split the component

u =  [tex]ln(x)[/tex]     du = [tex]\frac{1}{x}[/tex][tex]dx[/tex]

dv = [tex]x^{2}[/tex]         v = [tex]\frac{1}{3}x^{3}[/tex]

∫ u dv = uv - ∫ v du

[tex]x^{2} ln(x) dx[/tex]   =  ln(x) [tex]\frac{1}{3}x^{3}[/tex] - ∫ [tex]\frac{1}{3}x^{3}[/tex] [tex]\frac{1}{x}[/tex] dx

                    = [tex]\frac{1}{3}x^{3}[/tex] ln(x) - [tex]\frac{1}{3}[/tex] [tex]x^{3}[/tex] [tex]\frac{1}{x}[/tex] dx

                    = [tex]\frac{1}{3}x^{3}[/tex] ln(x) - [tex]\frac{1}{3}[/tex][tex]x^{3} . x^{-1}[/tex] dx

                    = [tex]\frac{1}{3}x^{3}[/tex] ln(x) - [tex]\frac{1}{3}[/tex][tex]x^{2}[/tex] dx     ----> [tex]x^{2}[/tex] dx = [tex]\frac{1}{3} x^{3} + C[/tex]

                    =  [tex]\frac{1}{3}x^{3}[/tex] ln(x) - [tex]\frac{1}{3}[/tex] [tex]\frac{1}{3} x^{3} + C[/tex]

                    = [tex]\frac{1}{3} x^{3} ( ln(x) - \frac{1}{3} ) + C[/tex]

Therefore Indefinite integral using integration by parts

[tex]x^{2} ln(x) dx[/tex]  is  [tex]\frac{1}{3} x^{3} ( ln(x) - \frac{1}{3} ) + C[/tex]

More about integration by parts on this link

https://brainly.com/question/1550271

#SPJ4

Jones Co. borrowed money that is to be repaid in 12 years. So that the loan will be paid back at end of the 12th year, the company invests $8,000 at end of each year at 5% compounded annually. The amount of the original loan was
Multiple Choice
$127,337.01
$70,905.60
$127,636.80
$70,950.60

Answers

The required future value of the annuity is $127,336.80.

Jones Company borrowed money that must be paid back in 12 years. To ensure that the debt is repaid by the end of the 12th year, the corporation invests $8,000 at the end of each year at a rate of 5% compounded yearly.

Compute the future value of the annuity as follows:

Future value = Annuity - FVIFA

FVIFA is taken from the table, and substitutes the value of the annuity in the above formula

Future value = $8,000 × 15.9171

Apply the multiplication operation, and we get

Future value = $127,336.80

Therefore, the required future value of the annuity is $127,336.80.

To learn more about the annuity click here:

brainly.com/question/13247656

#SPJ1


If I earn 100% more than you, then you must earn 50% less than me

Answers

Step-by-step brake down:
you: {number like 10} more than me
me: needs {half like 5} less than you

if you get 100% I would start with 0 so 0% + 50%

I don't know if I gave you what you need but I tried

A travel company has hired a management consultant company to analyze demand in twenty-six regional markets for one of its major products: a guided tour to a particular country. The consultant uses data to estimate the following equation:
Q=1500−4P+5A+10I+3PY
Where Q = amount of the product demanded;
P = price of the product in dollars;
A = advertising expenditures in thousands of dollars;
I = income in thousands of dollars;
PY = price of some other travel products offered by a competing travel company.
a) Calculate the amount demanded for this product using the following data:
P = $400
A = $20,000
I = $15,000
PY=$500
b) Suppose the competitor reduced the price of its travel product to $400 to match the price of this firm's product. How much would this firm have to increase its advertising in order to counteract the drop in its competitor's price? Would it be worth for them to do so? Explain your answer.
c) What other variables might be important in helping estimate the demand for this travel product?

Answers

Q falls to 251,400 to 201,400 which means a fall of 50,000 which should be counteracted by advertising. If advertising expenditure becomes 30,000 it will raise the Q again to 251,400.

Given that,

One of a travel company's main products, a guided tour to a specific country, was the subject of a demand analysis by a management consulting firm for 26 regional markets. The consultant estimates the following equation using data:

Q=1500−4P+5A+10I+3PY

Q is the quantity of the product that is demanded;

P represents the item's monetary price;

A represents advertising costs in thousands of dollars;

I represents income expressed in dollars;

PY is the cost of a different travel product provided by a rival travel agency.

We have to find

(a) Utilizing the information below, determine the amount demanded for this product:

P = $400

A = $20,000

I = $15,000

PY=$500

Q = 1,500 - 4 × 400 + 5 × 20,000 + 10 × 15,000 + 3 × 500

= 1,500 - 1,600 + 100,000 + 150,000 + 1,500

= 251,400

(b)  In Income falls to 10,000

Q = 1,500 - 4 × 400 + 5 × 20,000 + 10 × 10,000 + 3 × 500

= 1,500 - 1,600 + 100,000 + 100,000 + 1,500

= 201,400

Q falls to 251,400 to 201,400 which means a fall of 50,000 which should be counteracted by advertising. If advertising expenditure becomes 30,000 it will raise the Q again to 251,400.


To learn more about advertising visit: https://brainly.com/question/17761623

#SPJ4

S is the midpoint of RT. T has coordinates (-14,-15) and S has coordinates (11,-1) Find the coordinates of R

Answers

Answer:

(36, 13)

Step-by-step explanation:

to find the midpoint you have the add the x and the y and then divide by two

to find one of the endpoints you just do the opposite

you multiply the midpoint by 2

and then subtract the remaining endpoint from it

(11, -1) --> (22, -2)

now subtract T from it

(36, 13)

that is your answer

If the VO2 max test scores were normally distributed with a mean, median, and mode of 44ml/kg/min and standard deviation of 8, which of the following would be true. a.We should expect 16% of college students to score at least 36 on the VO2max test. b. We should expect 50% of college students to score 44 on the VO2max test. c. We should expect 84% of college students to score at least 36 on the VO2max test. d. We should expect at least 68% college students to score 36 on the VO2max test.

Answers

The VO2max test resulted by z score should result in a minimum score of 36 for 84% of college students.

To find the answer, we will use the z-score formula:

z = (x - μ)/σ

We will use the given values for the mean (μ = 44), the standard deviation (σ = 8), and the value we are looking for (x = 36).

z = (36 - 44) / 8

z = -1

We can then look up the percentage of values equal to or below -1 on the standard normal distribution table. This value is 84%. Therefore, we should expect 84% of college students to score at least 36 on the VO2max test.

Learn more about z score here

https://brainly.com/question/15016913

#SPJ4

The revenue and cost functions of a company are given as () = −5 2 + 750 and () = 100 + 20,000 respectively. (i) Determine the maximum revenue for the company (4 marks) (ii) By plotting the graph of () and () on the same axes, determine the break-even levels. (

Answers

The maximum revenue of the problem given at the end of the answer is of: $144.

$28,125.

The break-even point is of:

(50, 25000) and (80, 28000).

How to maximize revenue?

The revenue function is given as follows:

R(x) = -5x² + 750x.

Which is a quadratic equation with the coefficients given as follows:

a = -5, b = 750.

Then the x-coordinate of the vertex is of:

x = -b/2a = -750/-10 = 75.

Thus the maximum revenue is of:

R(12) = -5(75)² + 750(75) = $28,125.

What are the break-even points?

The break-even points are the points of intersection of the revenue and of the cost functions, given as follows:

Revenue: R(x) = -5x² + 750x.Cost: 100x + 20000.

From the intersection of the graph given at the end of the answer, these points are of:

(50, 25000) and (80, 28000).

More can be learned about maximizing revenue at https://brainly.com/question/13799721

#SPJ1


I need help with this please help me

Answers

The translation rule that maps A(-4, -2), B(-2, 6) and C (4, 4) to A' (-2, -2) B' (0, -10) and C' (6, -8) is

(x + 2, y + 4)

How to find he translation rule that suit the transformation

The transformation rule is solved by comparing the both preimage and image transformations as follows

preimage  image

A(-4, -2)   A' (-2, -2)

B(-2, 6)     B' (0, -10)

C (4, 4)     C' (6, -8)

x direction

-4 + x = -2, x = -2 + 4 = 2

-2 + x = 0, x = 0 + 2 = 2

4 + x = 6, x = 6 - 4 = 2

y direction reflection on the x axis leads to change of sign

preimage  image

A(-4, -2)   A' (-2, 2)

B(-2, 6)     B' (0, 10)

C (4, 4)     C' (6, 8)

-2 + y = 2, y = 2 + 2 = 4

6 + y = 10, y = 10 - 6 = 4

4 + y = 8, y = 8 - 4 = 4

The transformation rule is (x + 2, y + 4)

Learn more about transformation here:

https://brainly.com/question/29321312

#SPJ1

Function A is represented by the equation y= 2x+1. Function B is a linear function that goes through the points shown in the table. X 1 3 4 6 3 11 15 23 Which statement correctly compares the rates of change of the two functions?

A. the rate of change of function a is 1
The rat of change of function b is 4

B. the rate of change of function a is 2
the rate of change of function b is 8

C. the rate of change of function a is 1
the rate of change of function b is 8

D. the rate of change of function a is 2
the rate of change of function b is 4

Answers

The rate of change of the linear function A is 2, and the one of function B is 4, then the correct option is D.

Which statement correctly compares the rates of change?

A general linear function is written as:

y = a*x + b

Where a is the rate of change and b is the y-intercept.

In this case, we know that function A is:

y = 2x + 1

Then the rate of change is 2, and the y-intecept is 1.

We also know that if a line passes through two points (x₁, y₁) and (x₂, y₂), then the rate of change of that line is given by the formula:

a = (y₂ - y₁)/(x₂ - x₁)

In the case of function B we have the table:

x: 1   3  4  6

y: 3   11  15  23

We can use any two points on the table, if we use the first two ones:

(1, 3) and (3, 11) we will get:

a = (11 - 3)/(3 - 1) = 8/2 = 4

The rate of change of function B is 4, then the correct statement is the one in option D.

Learn more about linear functions:

https://brainly.com/question/4074386

#SPJ1

all the edges of a cube are expanding at a rate of 4 in. per second. how fast is the volume changing when each edge is 10in. long?

Answers

The rate at which the volume of the cube is changing is 1200 in³/seconds.

What is volume?

Volume is the space occupied by a solid object.

To calculate the rate at which the volume of the cube is changing, we use the formula below.

Formula:

dV/dt = (dL/dt)×(dV/dL)................ Equation 1

Where:

dV/dt = Rate at which the volume of the cube is changing dL/dt = Rate at which the edge of the cube is expandingdV/dL = Change in the volume of the cube with respect to the edge.

From the question,

Given:

dL/dt = 4 in. per secondsL = 10 in

If, the volume of a cube is V = L³,

Then,

dV/dL = 3L²  = (3×10²) = 300 in²

Substitute these values into equation 1

dV/dt = 4×300dV/dt = 1200 in³/seconds

Hence, the rate at which the volume is changing is 1200 in³/seconds.
Learn more about volume here: https://brainly.com/question/1972490

#SPJ1

DUE SOON!!! NEED HELP ON THESE TWO QUESTIONS PLEASE!
DUE SOON AND I DONT UNDERSTAND IT!
MANY THANKS!
WILL GET 100 POINTS! PLEASE HELP!!

Answers

The restrictions for x and y are given as follows:

3. x ∈ W, y ∈ W.

5. x ∈ W, y ∈ W.

What are the restrictions?

To obtain the restrictions, the first step is classifying the variables as either discrete or continuous, as follows:

Discrete variables are countable values, that can assume only whole or integer values, that is, they do not assume decimal values.Continuous variables can assume decimal values, hence they are real values.

In this problem, the variables are both resumed as follows:

Number of tickets sold in the Super Bowl.

The possible values are given as follows:

0, 1, 2, ...

Which are countable values, hence discrete, and that also cannot assume negative values, hence they belong to the whole set and not the integer set, thus the restrictions are:

x ∈ W, y ∈ W.

More can be learned about discrete and continuous variables at https://brainly.com/question/3130222

#SPJ1

sin(theta) * sec(theta) - 2sin(theta) = 0

Solve with all possible answers for theta
(please)

Answers

Answer:

θ = 2 π n_1 + π/3 for n_1 element Z

or θ = 2 π n_2 + (5 π)/3 for n_2 element Z

or θ = π n_3 for n_3 element Z

θ = π n_3 for θ element Z and cos(θ)!=0 and n_3 element Z

Step-by-step explanation:

Solve for θ:

tan(θ) - 2 sin(θ) = 0

Factor sin(θ) from the left hand side:

sin(θ) (sec(θ) - 2) = 0

Split sin(θ) (sec(θ) - 2) into separate parts with additional assumptions.

Assume cos(θ)!=0 from sec(θ):

sec(θ) - 2 = 0 or sin(θ) = 0 for cos(θ)!=0

Add 2 to both sides:

sec(θ) = 2 or sin(θ) = 0 for cos(θ)!=0

Take the reciprocal of both sides:

cos(θ) = 1/2 or sin(θ) = 0 for cos(θ)!=0

Take the inverse cosine of both sides:

θ = 2 π n_1 + π/3 for n_1 element Z or θ = 2 π n_2 + (5 π)/3 for n_2 element Z

or sin(θ) = 0 for cos(θ)!=0

Take the inverse sine of both sides:

θ = 2 π n_1 + π/3 for n_1 element Z

or θ = 2 π n_2 + (5 π)/3 for n_2 element Z

or θ = π n_3 for cos(θ)!=0 and n_3 element Z

The roots θ = π n_3 never violate cos(θ)!=0, which means this assumption can be omitted:

Answer: θ = 2 π n_1 + π/3 for n_1 element Z

or θ = 2 π n_2 + (5 π)/3 for n_2 element Z

or θ = π n_3 for n_3 element Z

Solve for θ over the integers:

tan(θ) - 2 sin(θ) = 0

Factor sin(θ) from the left-hand side:

sin(θ) (sec(θ) - 2) = 0

Split sin(θ) (sec(θ) - 2) into separate parts with additional assumptions.

Assume cos(θ)!=0 from sec(θ):

sec(θ) - 2 = 0 or sin(θ) = 0 for cos(θ)!=0

Add 2 to both sides:

sec(θ) = 2 or sin(θ) = 0 for cos(θ)!=0

Take the reciprocal of both sides:

cos(θ) = 1/2 or sin(θ) = 0 for cos(θ)!=0

Take the inverse cosine of both sides:

θ = 2 π n_1 + π/3 for θ element Z and n_1 element Z or θ = 2 π n_2 + (5 π)/3 for θ element Z and n_2 element Z

or sin(θ) = 0 for cos(θ)!=0

The roots θ = 2 π n_1 + π/3 violate θ element Z for all n_1 element Z:

θ = 2 π n_2 + (5 π)/3 for θ element Z and n_2 element Z

or sin(θ) = 0 for cos(θ)!=0

The roots θ = 2 π n_2 + (5 π)/3 violate θ element Z for all n_2 element Z:

sin(θ) = 0 for cos(θ)!=0

Take the inverse sine of both sides:

Answer: θ = π n_3 for θ element Z and cos(θ)!=0 and n_3 element Z

NO LINKS!!

f(x)= (x^2 + x - 12)/(x^2 + 6x + 9)

Discuss the behavior of f near any excluded x-values/
a. f(x) --> -∞ as x --> 3^+ and as x--> 4^-, f(x) --> ∞ as x --> 4^+ and as x --> 3^-
b. f(x) --> ∞ as x --> 4^+ and as x --> 4^-
c. f(x) --> -∞ as 4^+ and as x --> 4^-, f(x) --> ∞ as 3^+ and as x --> 3^-
d. f(x) --> -∞ as x --> 3^+ and as x --> 3^-, f(x) --> ∞ as x --> -4^+ and as x --> -4^-
e. f(x) --> -∞ as x --> -3^+ and as x --> -3^-

Answers

Answer:

  e. f(x) --> -∞ as x --> -3^+ and as x --> -3^-

Step-by-step explanation:

You want to know the behavior of f(x)= (x^2 + x - 12)/(x^2 + 6x + 9) near any excluded x-values.

Domain

The function can be factored as ...

  [tex]f(x)=\dfrac{x^2+x-12}{x^2+6x+9}=\dfrac{(x+4)(x-3)}{(x+3)^2}[/tex]

The excluded values are values of x where the denominator is zero. The only excluded value is x = -3. (eliminates all answer choices except E)

Asymptotic behavior

At either side of x = -3, the sign of the numerator is negative and the sign of the denominator is positive. That makes f(3-) < 0 and f(3+) < 0.

f(x) will never approach +∞, but f(x) approaches -∞ as x nears -3 from either direction.

Answer:

[tex]\textsf{e)} \quad f(x) \rightarrow -\infty\;\;\textsf{as}\;\; x \rightarrow -3^+ \;\;\textsf{and as}\;\;x \rightarrow -3^-[/tex]

Step-by-step explanation:

Given function:

[tex]f(x)=\dfrac{x^2+x-12}{x^2+6x+9}[/tex]

Factor the numerator:

[tex]\implies x^2+x-12[/tex]

[tex]\implies x^2+4x-3x-12[/tex]

[tex]\implies x(x+4)-3(x+4)[/tex]

[tex]\implies (x-3)(x+4)[/tex]

Factor the denominator:

[tex]\implies x^2+6x+9[/tex]

[tex]\implies x^2+3x+3x+9[/tex]

[tex]\implies x(x+3)+3(x+3)[/tex]

[tex]\implies (x+3)(x+3)[/tex]

[tex]\implies (x+3)^2[/tex]

Therefore, the rational function is:

[tex]f(x)=\dfrac{(x-3)(x+4)}{(x+3)^2}[/tex]

As the degree of the numerator is equal to the degree of the denominator, there is a horizontal asymptote at y = 1.

A vertical asymptote occurs at the x-value(s) that make the denominator of a rational function zero.

[tex]\implies (x+3)^2=0[/tex]

[tex]\implies x+3=0[/tex]

[tex]\implies x=-3[/tex]

Therefore, there is a vertical asymptote at x = -3.

As there is a vertical asymptote at x = -3, the excluded x-value is x = -3.

As x approaches x = -3 from both sides, the numerator of the rational function approaches -6 and the denominator approaches a very small positive number.  Therefore, the function approaches a very large negative number.

Therefore, the end behaviour of the function as it approaches the excluded value is:

[tex]f(x) \rightarrow -\infty\;\;\textsf{as}\;\; x \rightarrow -3^+ \;\;\textsf{and as}\;\;x \rightarrow -3^-[/tex]

assume that a and b are positive integers. for each statement below, determine whether the statement is true or false, and indicate your answer in the appropriate box

Answers

Given Statement (a!)b = ab! is false becuase it is not appropriate to represent factorial like this.

What is Factorial?

In mathematics, the term "Factorial" refers to the sum of all positive integers that are less than or equal to a certain positive integer, which is followed by an exclamation point. Thus, factorial seven is denoted by the symbol 7!, which stands for 1*2*3*4*5*6*7. Factorial 0 is equivalent to 1 by definition.

(a!)*b = ab!

this depends on the value of a and b.

I'm not sure if the phrase "ab!" means to multiply a by b and then compute the factorial of the resulting product, as in (ab)!, or to compute the factorial of b first and then multiply this by a, as in a(b!). Either way, the assertion is false.

Given Question is incomplete Complete Question here:

assume that a and b are positive integers. for each statement below, determine whether the statement is true or false, and indicate your answer in the appropriate box  (a!)b = ab!  

To know more about factorial here:

https://brainly.com/question/25505833

#SPJ4

When you are doing a survey of the time it takes for a person to run 100m, which unit would you use?

Answers

When you are doing a survey of the time it takes for a person to run 100m, the unit that you would you use is seconds.

What is unit?

A unit of measurement is a definite magnitude of a quantity that is defined and adopted by convention or law and is used as a standard for measuring the same type of quantity. Any other such quantity can be expressed as a multiple of the unit of measurement. A length, for example, is a physical quantity.

A unit is any standard that is used to compare measurements. Unit conversions allow measurements of a property to be converted from one unit to another, such as centimeters to inches.

In this case, time is measured in seconds.

Learn more about unit on:

https://brainly.com/question/28464

#SPJ1

where is root 19??????

Answers

Answer:

4.358

Step-by-step explanation:

sqrt(19) = 4.358

john is jogging on a straight jogging trail toward a flagpole at a speed of 500 ft/min. he is currentyl 1500 feet from the flagpole.

Answers

As per the unitary method, it take 3 minutes will it take John to reach the flagpole.

The term unitary method in math is defined as in which the value of one article is first obtained to find out the value of any required articles.

Here we have given that John is jogging on a straight jogging trail toward a flagpole at a speed of 500 ft/min. He is currently 1500 feet from the flagpole. And here we have to write an absolute value function that models John's distance from the flagpole after x minutes.

Here let us consider x be the time taken by John to reach the flagpole.

And here we know that

speed = 500 ft/min

height = 1500 feet

Then the absolute function for the given situation is written as,

=> f(x) = 500|x|

Apply the value of f(x) as 1500, then we get

=>  1500 = 500|x|

=> x = 3

To know more about Unitary method here.

https://brainly.com/question/28276953

#SPJ4

A political action committee sends out a questionnaire to randomly-selected mailing addresses, asking people to rate the importance of a variety of economic and social issues facing the country. At the end of the questionnaire is an (optional) invitation to donate money to the organization. Which of the following statements about this survey is true?
A) The survey results are invalid because it’s impossible to word questions about politics without creating bias.

B) The survey results are invalid because only surveys conducted by phone are free from bias.

C) The survey results will underestimate support for this political action committee in the entire population because people who respond are less likely to support an organization that asks for money.

D) The survey results will overestimate support for this political action committee in the entire population because people more likely to respond if they are prepared to donate money.

E) Since the survey was sent to randomly-selected households, it should be free of bias.

Answers

Option c; The survey results will underestimate support for this political action committee in the entire population because people who respond are less likely to support an organization that asks for money. This is the correct statement about the survey.

What is politics?

Politics is a category of activities that are related to group decision-making or other types of power dynamics between people, such as the allocation of resources or status. Political science is the name of the area of social science that focuses on politics and governance.

Given,

A political action committee sends out a questionnaire to randomly-selected mailing addresses, asking people to rate the importance of a variety of economic and social issues facing the country. at the end of the questionnaire is an (optional) invitation to donate money to the political action committee.

We have to find the correct statement about the survey from the given options;

Because respondents are less inclined to support a group that solicits donations, the survey results will understate public support for this political action committee.

The correct statement about the survey is option c;

Learn more about political action committee here;

brainly.com/question/26270345

#SPJ1

In Norway last year, the lowest temperature was −15°C. In Norway last year, the highest temperature was 42°C greater than the lowest temperature. Work out the highest temperature in Norway last year

Answers

Answer:

27°

Step-by-step explanation:

Let h = highest temperature

h = -15 + 42

h = 27°

Find the Maclaurin series for f(x) = xe^2x and its radius of convergence. Use only the definition of a Maclaurin series.

Answers

The Maclaurin Series of xe²ˣ: x + 2x² + 2x³+ 4/3x⁴+ 2/3x⁵ +....

Given that,

f(x) = xe^2x

Taylor series of function f(x) at a is defined as:

f(x) = f(a) + f'(a)/1!/(x-a) +f''(a)/2! (x-a)² +....

Maclaurin series of Function F(x) is a Taylor series of Function f(x) at: a=0

f(x) = f(0) + f'(0)/1! /(x) +f''(0)/2!/ (x)² +....

Apply the Maclaurin Formula

= 0 +d/dx (xe²ˣ)(0)/1! x + d²/dx² (xe²ˣ)(0)/2! x² +....

Evaluate the derivatives :

= 0 + 1/1! x + 4/2! x² + 12/3! x³ + ...

Refine :

= x + 2x² + 2x³+ 4/3x⁴+ 2/3x⁵ +....

So, the Maclaurin series for f(x) = xe^2x is  x + 2x² + 2x³+ 4/3x⁴+ 2/3x⁵ +....

To learn more about Maclaurin Series click here:

brainly.com/question/24188694

#SPJ4

The price of a gift plus 14% delivery charge comes to a total cost of $19.38. What was the price of the gift?

Step 1 of 2 : Describe the above situation as a linear equation, using "x" or "y" as variable names to describe the unknown quantity.

Answers

Answer:

1.14x = 19.38

$17

Step-by-step explanation:

Let the price before delivery charge = x.

The charge is 14%, so it is 14% of x, or 0.14x.

The original price plus charge is x + 0.14x, or 1.14x.

1.14x = 19.38

x = 19.38/1.14

x = 17

The price is $17

Other Questions
what is twice the area of the surface formed by points (1,0,0),(0,2,0),(0,0,3)? What are the similarities between the characters Citra and Rowan from Scythe? chris anderson, editor-in-chief of wired magazine, describes niche-market ebusiness strategies as capturing the tail, referring to the tail of a typical sales curve. multiple choice question. Which of the following steps must be completed before preparing the adjusted trial balance?A. Preparation of the financial statementsB. Preparation of the closing entriesC. Posting of journal entries to the accountsD. Preparing the post-closing trial balance What is protagonist or antagonist meaning? Blue Corporation purchased trading investment bonds for $55,000 at par. At December 31, Blue received annual interest of $2,200, and the fair value of the bonds was $52,500. Prepare Blue's journal entries for (a) the purchase of the investment, (b) the interest received, and (c) the fair value adjustment Saved Drag each of the following labels into the appropriate box to identify which motor division of the peripheral nervous system is described Associatedh sensory Autonomic Somatic receptors for touch Only requires one neuron from the CNS to the effector organ Requires two neurons from the CNS to the effector organ Utilizes acetylcholine and norepinephrine as neurotransmitters Presence of pre- and post- ganglionic neurons Associated with voluntary motor neurons CNS somas are located in the lateral homs of the spinal cord Reset what evidence does the author give to support his claim that long walks are good for personal well-being? select several friends' concurring opinions select a personal anecdote select scientific research select he provides no evidence in favor of his argument. select health statistics Which of the following is not a determinant of an organization's structure? a strategy b. organization sizeC. industry d. technology which of the following occurs naturally as nonpolar diatomic molecules?A. HydrogenB. NitrogenC. OxygenD. All of the aboveE. None of the above which fire suppression system class is required for protecting server rooms full of computing equipment? dorsal and ventral rami are similar in that they both contain sensory and motor fibers. Why is an electric current only produced if there is a closed circuit?Without a closed circuit, electrons cannot flow from one point to another. A closed circuit allows magnets to move back and forth, creating an electric current. An open circuit causes electrons to escape the wire, stopping the electric current. With an open circuit, the magnetic field created would cause the current to stop flowing. Fred was assigned the task of creating a new group on the company Linux server and now needs to assign permissions for that group to files and directories. What Linux commands should he use to change the group assigned to the files and directories? A person places $8620 in an investment account earning an annual rate of 6.1%, compounded continuously. Using the formula V = Pe^{rt}V=Pe rt , where V is the value of the account in t years, P is the principal initially invested, e is the base of a natural logarithm, and r is the rate of interest, determine the amount of money, to the nearest cent, in the account after 14 years. thanks!! 28. Which method or operator can be used to concatenate lists? A. B. + C. % D. concat 29. What will be the value of the variable list after the following code executes? list = (1, 2, 3, 4] list[3] = 10 A. [1, 2, 3, 10] B. [1, 2, 10, 4] C. [1, 10, 10, 10] D. Nothing; this code is invalid 30. Consider the following list: inventory = ["washer", "dryer", "microwave") Which of the following will display the item value, "dryer"? A. print(inventory[0]) B. print(inventory(1) C. print(inventory[2]) D. printinventory(-1)) 31. What will be the value of the variable list2 after the following code executes? list1 = [1, 2, 3] list2 = 0 for element in list1: list2.append(element) list1 = [4, 5, 6] A. [4, 5, 6] C. (1,2,3,4,5,6] B. (1, 2, 3] D. None of the above 32. Given that: aList = [1,2,3,4] which of the following will display the list [2,3,4] at the screen? A. print(aList[1:3]) B. print(aList[2:41) C. print(aList[2:1) D. print(aList[-3:]) 33. Given that: aList = [1,2,10,11,12,13] del aList[3] what will be printed by print (aList) A. [1,2,11,12,13] C. [1,2,10,12,13] B. (1,10,11,12,13] D. (1,2,10,11,13] 34. Given that: my_tools = ['hammer', 'screwdriver', 'wrench', 'pliers', 'nailer', 'table saw') my_tools[2:4]= ['chainsaw', 'chipper'] what will be printed by print (my_tools) A. ['hammer','chainsaw', 'chipper','screwdriver','wrench','pliers', 'nailer', 'table saw'] B. ['hammer','screwdriver', 'wrench', 'chainsaw', 'chipper', 'nailer', 'table saw') C. ['hammer','screwdriver', 'chainsaw','chipper', 'nailer', 'table saw'] D. ['hammer',chainsaw', 'chipper','pliers', 'nailer', 'table saw') 35. Given the following list: Names = ['Dan', 'Mary', 'Bill', 'Juanita', 'Harry', 'Sarah'] Which of the following will change the list names to ['Dan', 'Mary', Juanita', 'Harry', 'Sarah') ? A. del Names[3] B. erase('Bill') C. Names.pop('Bill') D. Names.pop(-4) E. Names.insert(2, nothing) if there appears to be a relationship in scatterplot 4, would you describe the linear relationship as positive or negative? an experiment is conducted in which red light is diffracted through a single slit. part a listed below are alterations made, one at a time, to the original experiment, and the experiment is repeated. after each alteration, the experiment is returned to its original configuration. which of these alterations decreases the angles at which the diffraction minima appear? select all that apply. view available hint(s)for part a listed below are alterations made, one at a time, to the original experiment, and the experiment is repeated. after each alteration, the experiment is returned to its original configuration. which of these alterations decreases the angles at which the diffraction minima appear? select all that apply. the experiment is conducted in a water-filled tank. the distance between the slits and the screen is halved. a green, rather than red, light source is used. the slit width is doubled. the slit width is halved. the distance between the slits and the screen is doubled. Which of the following solution in water will have the highest freezing point?Option 1 Solution which is 0.1 M in KNO3 and 0.2 M in Ba(NO3)2. Option 2 Solution which is 0.1 M in Ca(NO3)2 and 0.1 M Ba(NO3)2. Option 3 Solution which is 0.2 M in urea and 0.2 M glucose. Option 4 Solution which is 0.1 M both in Al2(SO4)3 and 0.1 M K2SO4. A: Al2O3B: Cr2O3C: V2O5D: MnO2 why did the united states stay neutral at first, and what made it enter world war i?